Time of approach of two oppositely charged particle

In summary: Please help. The kinetic energy of the system is equal to the kinetic energy of the charge plus the kinetic energy of all the particles relative to the charge. In summary, the two point charges will collide in seconds if they are at a distance of 1 meter. The charge at the left is negative so the collision will happen to the right. The velocity of the particles will increase as the distance between the charges decreases.
  • #1
Satvik Pandey
591
12

Homework Statement



Two point charges q1=1[itex]\mu[/itex]C and q2=-1[itex]\mu[/itex]C with mass 1g each are at distance 1m from each other. The charges are released from rest at . Find the time t in seconds at which they collide.

Homework Equations


F=kQ1*Q2/r^2.

The Attempt at a Solution


Let the dist. of separation be r and the mass of charged be m.
I think they will meet at the middle of separation.Force on q1=kq^2/r^2 so the acceleration =kq^2/r^2*m.But in this case acceleration is not constant.So I have to use calculus here.
Let the charge q1 be at origin and at a distance x from origin acceleration=kq^2/(r-2x)^2*m( 'r-2x' because if q1 will move x meter towards q2 then q2 will also move x metre towards q1 so the length of separation will decrease by 2x) .
For a small displacement dx this acceleration is constant.I tried to use S=ut+at^2/2.But I am unable to find u.Please help.
 
  • Like
Likes 1 person
Physics news on Phys.org
  • #2
Have you studied differential equations?

When setting up the solution, draw a picture first and show how you define the variables in that picture.

The particles move symmetrically with respect to the centre. If one is at x m distance from the centre, how far is the other? And what is the distance between them?

ehild
 
  • #3
ehild said:
The particles move symmetrically with respect to the centre. If one is at x m distance from the centre, how far is the other? And what is the distance between them?

ehild

The other should also be x m distance from the center.So the distance between them should be 2x.
 
  • #4
I should have choose origin at the center.Now at a distance x from the origin acceleration of q1 will be kq^2/(4x^2*m).Here m is for mass of particle.
I have made this figure
Physics problem.png
.
 
  • #5
Satvik Pandey said:
I should have choose origin at the center.Now at a distance x from the origin acceleration of q1 will be kq^2/(4x^2*m).Here m is for mass of particle.
I have made this figure View attachment 71218.

Good. How the acceleration is related to the second derivative of x?

ehild
 
  • #6
ehild said:
Good. How the acceleration is related to the second derivative of x?

ehild

[itex]\frac{d^{2}x}{dt^{2}}[/itex]=acceleration
 
  • #7
Write the equation of motion ma=F in terms of x. You wrote that the acceleration is kq^2/(4mx2). It is positive. That means x increasing with time. Is it true? Write the equation of motion in terms of x.

ehild
 
Last edited:
  • #8
ehild said:
Write the equation of motion ma=F in terms of x. You wrote that the acceleration is kq^2/(4mx2). It is positive. That means x increasing with time. Is it true? Write the equation of motion in terms of x.

ehild

kq[itex]^{2}[/itex]/4x[itex]^{2}[/itex]=m*d^2x/dt^2
I think x decreases with time because as the time passes both the particle move toward origin so their distance of separation decreases.
 
  • #9
Look at m1. The force acts to the right. The acceleration is to the right. Does x increase?

ehild
 
  • #10
According to figure which I have made in post#4 it seems that x decreases and acceleration increases because x is measured from origin and in this figure origin is at the middle of the line of separation.
 
  • #11
Yes. So -kq2/4x2=m*d^2x/dt^2
That is a differential equation, it is not easy to solve. You can go one step ahead by applying Conservation of Energy. Then you get the speed as function of x. Try.
ehild
 
  • #12
ehild said:
Yes. So -kq2/4x2=m*d^2x/dt^2
That is a differential equation, it is not easy to solve. You can go one step ahead by applying Conservation of Energy. Then you get the speed as function of x. Try.
ehild

I have not applied conservation of energy concept to system of particles earlier so this is my first time.
Initially E(potential)=-[itex]\frac{kq^{2}}{r}[/itex] (I got -ve sign as the charge at the left is negative)
Now at x kinetic energy of system = kinetic energy of the CM+ kinetic energy of all particles relative to the CM.
E[itex]_{k}[/itex] at x=mv[itex]^{2}[/itex]
E[itex]_{p}[/itex] at x = -[itex]\frac{kq^{2}}{2x}[/itex]
so
-[itex]\frac{kq^{2}}{r}[/itex]=mv[itex]^{2}[/itex]+ -[itex]\frac{kq^{2}}{2x}[/itex].

v=sq.root of [itex]\frac{kq^{2}(r-2x)}{2mxr}[/itex].
the figure is here View attachment 71218.V is the velocities of particles at x.
 
Last edited:
  • #13
Now write v =dx/dt and integrate to find x as function of time .
 
  • #14
Be careful with the signs. x is the distance between the charges. It decreases with time, dx/dt<0. v is the speed of both particles, a positive quantity.

ehild
 
  • #15
Well I got
-[itex]\frac{√2x}{√(1-2x)}[/itex]dx=3*10^6 dt.
I have some problem in solving LHS.I substituted 2x=a^2.
And I got dx=ada.On putting values I got
[itex]\frac{a^2}{√(1-a^2}[/itex]da.
Don't know what to do next.
 
Last edited:
  • #16
This will not work . Try some trigonometric substitution and see which one fits in.
 
  • #17
Tanya Sharma said:
This will not work . Try some trigonometric substitution and see which one fits in.

If denominator is in the √a^2-x^2 and numerator is dx then we substitute x as asinθ.I don't have very much knowledge of calculus.I know only some basics.Will the substitution work.
 
  • #18
While performing integration using substitution method you need to substitute values of both x as well as dx in terms of θ.

For example if you are using the substitution x= asinθ , then dx = (acosθ)dθ .
 
Last edited:
  • #19
Satvik Pandey said:
Well I got
-[itex]\frac{√2x}{√(1-2x)}[/itex]dx=3*10^6 dt.

You can cheat a bit... It is not an easy integral. Use wolframalpha.com

Check the right-hand side, it does not look correct.

http://www.wolframalpha.com/input/?i=int_0.5^0%28sqrt%282x%2F%281-2x%29%29dx%29

ehild
 
Last edited:
  • #20
Yes . The integral is not easy .
 
Last edited:
  • #21
Tanya Sharma said:
While performing integration using substitution method you need to substitute values of both x as well as dx in terms of θ.

For example if you are using the substitution x= asinθ , then dx = (acosθ)dθ .

Well I substituted 2x=sin[itex]^{2}[/itex]θ.
so dx=sinθcosθdθ.
On substituting the values
√(sin^2θ)/√(1-sin^2θ) dθ

=sin^2θ dθ

=θ/2 - sin2θ/4
 
  • #22
Very nice! Now the limits. What are the limits of integration?

ehild
 
  • #23
ehild said:
Check the right-hand side, it does not look correct.


ehild

[itex]\frac{dx}{dt}[/itex]=[itex]\frac{kq^{2}(r-2x)}{2mxr}[/itex]

On putting the values I got RHS as

9*10^9*1(1-2x)/2*10[itex]^{-3}[/itex]*x*1
( the mass was given in grams and that has to converted into kg)
so dx/dt=( 9* 10^12(1-2x)/2x)

so I got this ∫√2x/√(1−2x)dx=∫3*10^6 dt.
 
  • #24
I meant the constant sqrt(kq^2/(mr)). You seem to omit q^2. q=1 μC.

ehild
 
  • #25
ehild said:
Very nice! Now the limits. What are the limits of integration?

ehild

Earlier the limits of x are from o.5 to 0 but 2x=sin^2 θ
so now the limits are from 90 degrees to 0
 
  • #26
Yes, but you should use radians.

ehild
 
  • #27
ehild said:
I meant the constant sqrt(kq^2/(mr)).

ehild

Was the equation in post#12 not correct?
 
  • #28
ehild said:
Yes, but you should use radians.

ehild

So the limits are from 'pi'/2 to 0
 
  • #29
Satvik Pandey said:
Was the equation in post#12 not correct?

I was correct but that in post #15 was wrong. The number in the right-hand side.

ehild
 
  • #30
Satvik Pandey said:
So the limits are from 'pi'/2 to 0

Excellent!

ehild
 
  • #31
ehild said:
I meant the constant sqrt(kq^2/(mr)). You seem to omit q^2. q=1 μC.

ehild

Yes I forgot that q=1 μC.
It should be ∫√2x/√(1−2x)dx=∫3 dt.
 
  • #32
Yes. So what is t then?

ehild
 
  • #33
ehild said:
Yes. So what is t then?

ehild

LHS is 0.785398 and RHS is 3t

So I got t=0.2619sec but the answer is 0.74second.
Is the answer correct?
 
  • #34
I think your solution is correct.

ehild
 
  • Like
Likes 1 person
  • #35
I am also getting ##\frac{\pi}{12} sec## as the answer .
 
  • Like
Likes 1 person

Similar threads

  • Introductory Physics Homework Help
Replies
9
Views
1K
  • Introductory Physics Homework Help
Replies
3
Views
1K
  • Introductory Physics Homework Help
Replies
9
Views
3K
  • Introductory Physics Homework Help
Replies
2
Views
1K
  • Introductory Physics Homework Help
Replies
5
Views
3K
  • Introductory Physics Homework Help
Replies
2
Views
1K
  • Introductory Physics Homework Help
Replies
1
Views
712
  • Introductory Physics Homework Help
Replies
1
Views
885
  • Introductory Physics Homework Help
Replies
2
Views
7K
  • Introductory Physics Homework Help
Replies
4
Views
1K
Back
Top